r(t) =

t2 − 36
i − j + ln(t − 1)k
(a)
Find the domain of r.

Answers

Answer 1

The domain of the vector function is : ( -6, -2) ∪ ( -2, 6)

The given vector function can be correctly expressed as:

r(t) = (t-2)i / (t+2) + sintj + ln(36-t²)k²

The domain of the given function can be determined by finding the domain of each respective component.

(t-2)i / (t+2)

Not defined, t = -2

Thus, the domain of the first component of the vector = (-∞, 2) ∪ (2, ∞)

The 2nd component = sin t

No restriction on t,

The 3rd component of the function is ㏑(36 - t²)

we know,

Natural number is defined for +ve numbers,

i.e.

36 - t² > 0

(6 + t) (6 - t) > 0

thus, it satisfies the inequality -6 < t < 6

The third domain = (-6,6)

Hence, the domain of the vector function is : ( -6, -2) ∪ ( -2, 6)

Learn more about vector function click;

https://brainly.com/question/31473715

#SPJ1


Related Questions

Margarita fue a la tienda y compro una cartera y unos jeans por un
total de RD$3,250. Sabiendo que las cartera excede al jeans en
RD$970, ¿Cuántos pago margarita por cada artículo?
Cartera = RDS
Jeans = RD$
Escribir las respuestas numéricas y sin comas.
OK

Answers

The solution is , price of jeans = RD$ 1140 and, price of  handbag =

RD$ 2110.

Here, we have,

given that,

Margarita went to the store and bought a bag and some jeans for a total of RD$3,250.

Knowing that the handbag exceeds the jeans by RD$970,

now, we have to find that, how many do she pay for each article.

let, price of jeans = RD$ x

so, price of  handbag = RD$ (x +970)

ATQ, we get,

RD$ x + RD$ (x +970) = RD$3,250

or, RD$ 2x + 970 = RD$3,250

or, RD$ 2x = RD$ 2280

or, x = RD$ 1140

Hence, price of jeans = RD$ 1140 and, price of  handbag = RD$ 2110.

To earn more on addition click:

brainly.com/question/29560851

#SPJ1

Determine if the statement is true or false.

Angle AEC and DEB are complementary angles and therefore add up to 180 degrees.

1. True
2. False

Answers

The statement "Angle ∠AEC and ∠DEB are complementary angles" is false.

Supplementary angle - Two angles are said to be supplementary angles if their sum is 180 degrees.

Complementary angle - Two angles are said to be complementary angles if their sum is 90 degrees.

Angle ∠AEC and ∠DEB are supplementary angles because the sum is 180 degrees.

Thus, the given statement is false.

More about the angled link is given below.

https://brainly.com/question/15767203

#SPJ1

Help with this page :)

Answers

17. 15 degrees

18. ABC

19. 5.88

20. 11.59

let me know if you'd like an explanation

A group consists of seven Democrats and eight Republicans. Four people are selected to attend a conference.
a. In how many ways can four people be selected from this group of fifteen?
b. In how many ways can four Republicans be selected from the eight Republicans?
c. Find the probability that the selected group will consist of all Republicans.
a. The number of ways to select four people from the group of fifteen is
b. The number of ways to select four Republicans from the group of eight Republicans is
c. The probability is

Answers

There 1365 ways to choose four people from the group of fifteen.

b. There are 70 ways to choose  four Republicans from the group of eight Republicans.

C. The probability is about  0.0513, or  5.13%.

What is the probability  about?

a.  To know the ways that four people can be selected from this group of fifteen is by:

nCr = n! / (r! x (n-r)!),

Where:

n  = total number of items

r = is the number of items to be selected,

!  = the factorial of a number.

Putting in the values into the the formula:

15C4 = 15! / (4! x (15-4)!)

(15-4)! = 11!

15C4 = 1365

B.  Since:

n = 8

r = 4

Putting in the values into the the formula:

8C4 = 8! / (4!  x (8-4)!)

(8-4)! = 4!

8C4 = 70

c. The  Probability = Number of ways to choose four Republicans / Number of ways to choose four people

Hence  Probability = 70 / 1365

                             =  0.0513

Therefore, the probability that the selected group will consist of all Republicans is about  0.0513, or  5.13%.

Learn more about probability  from

https://brainly.com/question/24756209

#SPJ1

geometry pls help fast 13 and 14

Answers

Answer:

13: (2,3) 14: A. Yes, 90, 180, 270 B. No C. No

Step-by-step explanation:

13. Right: -2+4, Down: 5-2

14. If it can be rotated and be similar, it has rotational.

Please help asap!!!!! I'm confused

Answers

The area of the parallelogram is 8.5 square miles. This is found by multiplying the length of one of the parallel sides, 2 miles, by the height, which is given as 4 1/4 miles.

To find the area of a parallelogram, we can multiply the length of one of its parallel sides by the length of its perpendicular height. Therefore, to find the area of this parallelogram, we need to determine its height.

We are given that one of the parallel sides has a length of 4 1/4 mi and the other has a length of 2 mi. We are also given that the length of the perpendicular on one of the parallel sides is 4 1/4 mi, which means that this is the height of the parallelogram.

So, the area of the parallelogram is

Area = base x height

Area = 2 mi x 4 1/4 mi

Area = 8 1/2 mi²

Therefore, the area of the parallelogram is 8 1/2 square miles or 8.5 square miles.

To know more about Area of parallelogram:

https://brainly.com/question/19187448

#SPJ1

Keng adds a 3-inch-wide frame around all sides of his canvas. ​

Answers

Answer:

That statement describes an action taken by Keng to add a 3-inch-wide frame around all sides of his canvas, likely for artistic or aesthetic purposes. This would result in the canvas being extended by 3 inches in each direction, effectively increasing its overall dimensions and adding a border or frame around the original artwork. The purpose and effect of adding a frame may vary depending on the artistic intention of the artist, but it is a common practice in art and design to enhance the presentation and visual appeal of the artwork.

Step-by-step explanation:

Find the angle between the pair of vectors to the nearest tenth of the degree

Answers

The value of angle between the two vectors is 86⁰.

What is the angle between the two vectors?

The value of angle between the two vectors is calculated as follows;

tan θ = vy/vx

where;

vy is the sum of the vertical directionvx is the sum of vectors in horizontal direction

( -8, 9), (-9, -6)

vy = (-6 - 9) = -15

vx = (-9 + 8) = -1

tan θ = ( -15 ) / ( -1 )

tan θ = 15

The value of θ is calculated  by taking arc tan of the fraction,;

θ = tan ⁻¹ ( 15 )

θ =  86⁰

Learn more about direction here: https://brainly.com/question/30318208

#SPJ1

In Brian's grade, 440 students are enrolled in health and 60 students are not. What percentage of the students in the school are enrolled in health?

Answers

88% of the students in the school are enrolled in health.

We have,

Students enrolled in health = 440

Students not enrolled = 60

Total students = 500

So, x% of 500 = 440

x/100 (500) = 440

5x = 440

x = 88%

Thus, 88% of the students in the school are enrolled in health.

Learn more about Percentage here:

https://brainly.com/question/29306119

#SPJ1

Need help. This please

Answers

The domain of the quadratic function in this problem is given as follows:

All real values.

How to obtain the domain of the function?

The domain of a function is the set of all the possible input values that can be assumed by the function.

On the graph, the domain of the function is given by the values of x of the function.

A quadratic function has no restrictions on the domain, hence it is defined by all the real values.

More can be learned about the domain of a function at https://brainly.com/question/10687170

#SPJ1

Question 1 (1 point)
Write an inequality for the sentence.
The stadium held less than 25,000.
B
O a
O b
9ba580e611107c96c9efb866417dc160.webm 64 KB
s> 25,000
$≤25,000
Oc $<25,000

Answers

The inequality that represents the sentence "The stadium held less than 25,000 people" is given as follows:

c < 25,000.

What are the inequality symbols?

The four inequality symbols, along with their meaning on the number line and the coordinate plane, are presented as follows:

> x: the amount is greater than x -> the number is to the right of x with an open dot at the number line. -> points above the dashed horizontal line y = x on the coordinate plane.< x: the amount is less than x. -> the number is to the left of x with an open dot at the number line. -> points below the dashed horizontal line y = x on the coordinate plane.≥ x: the amount is at least x. -> the number is to the right of x with a closed dot at the number line. -> points above the solid vertical line y = x on the coordinate plane.≤ the amount is at most x. -> the number is to the left of x with a closed dot at the number line. -> points above the dashed vertical line y = x on the coordinate plane.

The amount is less than in this problem, hence the symbol is given as follows:

<.

As the amount is less than 25000, the inequality is given as follows:

c < 25,000.

More can be learned about inequalities at brainly.com/question/25275758

#SPJ1

Which equation represents a direct variation?
Oy - 4x = 8
Oy + 2 = 7x
Oy - 3x = 0
O y = 5x - 2

Answers

The only equation that represents a direct variation is: y - 3x = 0

How to identify direct variation?

Direct Variation is defined as the relationship that exists between two variables in which one is a constant multiple of the other. For example, when one variable changes the other, then they are said to be in proportion. If b is directly proportional to a the equation is of the form b = ka (where k is a constant).

Making y the subject in each of the options gives:

A) y = 4x + 8

B) y = 7x - 2

C) y = 3x

D) y = 5x - 2

Looking at them, the only one where there is a relationship that exists between two variables in which one is a constant multiple of the other is option C

Read more about Direct variation at: https://brainly.com/question/6499629

#SPJ1

use the equation 1/5 +s =32/40

Answers

The required solution to the equation 1/5 + s = 32/40 is s = 3/5.

To solve the equation 1/5 + s = 32/40 for s, we can begin by subtracting 1/5 from both sides to isolate s:

1/5 + s = 32/40

s = 32/40 - 1/5

We need a common denominator to combine the fractions on the right side of the equation. The least common multiple of 5 and 40 is 40, so we can convert both fractions to have a denominator of 40:

s = (32/40) - (8/40)

s = 24/40

Simplifying the fraction 24/40 by dividing both the numerator and denominator by their greatest common factor, which is 8, we get:

s = 3/5

Therefore, the solution to the equation 1/5 + s = 32/40 is s = 3/5.

Learn more about simplification here:

https://brainly.com/question/12501526

#SPJ1

Brooklyn and Matthew both recently got a job and want to start a savings account to earn interest on money they save. Brooklyn's paycheck for was for $625 net pay-after taxes. the bank her parents use offer savings account compounded monthly with an interest rate of 2.5%. Matthews paycheck was also for $446 net pay. his parents know of a savings account that earns 4% compounded quarterly.
a. use the compounding interest rate formula to identify who will have more money after two years (assuming no additional deposits or withdrawals are made)
b. use the Desmos graphing calculator to graph Brooklyn's equation and Matthew’s equation on the same graph. identify the points of intersection and interpret the results.

WHAT MISTAKES DID THE STUDENTS MAKE??

Answers

After two years, Brooklyn will have more money in her savings account than Matthew.

Brooklyn's savings account grows at a slower rate than Matthew's initially but eventually catches up and surpasses Matthew's account due to the higher interest rate and monthly compounding.

a.

[tex]A = P(1 + r/n)^{nt}[/tex]

where A is the final amount, P is the initial principal (or net pay), r is the interest rate (as a decimal), n is the number of times the interest is compounded per year, and t is the number of years.

For Brooklyn saving account,

P = $625, r = 0.025, n = 12 (monthly compounding), and t = 2.

A = $625(1 + 0.025/12)^(12*2) = $686.71

For Matthew saving account,

P = $446, r = 0.04, n = 4 (quarterly compounding), and t = 2.

A = $446(1 + 0.04/4)^(4*2) = $506.84

b.

To graph Brooklyn's equation and Matthew's equation on the same graph, we can use the following equations:

Brooklyn's equation: A = 625(1 + 0.025/12)^(12t)

Matthew's equation: A = 446(1 + 0.04/4)^(4t)

Now,

The resulting graph will show the growth of each saving account over time.

The points of intersection on the graph represent the times when the two savings accounts have the same amount of money.

From the graph, we can see that the two accounts intersect at around 11 months and 23 months.

We can say that Brooklyn's savings account grows at a slower rate than Matthew's initially, but eventually catches up and surpasses Matthew's account due to the higher interest rate and monthly compounding.

Thus,

After two years, Brooklyn will have more money in her savings account than Matthew.

Brooklyn's savings account grows at a slower rate than Matthew's initially but eventually catches up and surpasses Matthew's account due to the higher interest rate and monthly compounding.

Learn more about saving accounts here:

https://brainly.com/question/13155407

#SPJ1

A geography teacher assigns each student to write a report about one of the first 13 colonies. Students select the name of a colony by "blindly" drawing a colony's name from a bag. Once a colony has been drawn, it is not replaced.

What is the probability that the first student selects Pennsylvania and the second student selects Virginia? Round your answer to the nearest hundredth of a percent.

Answers

The probability that the first student selects Pennsylvania and the second student selects Virginia to the nearest hundredth of a percent is 0.64%.

Probability problem

The probability that the first student selects Pennsylvania is 1/13. Once Pennsylvania has been drawn, there are only 12 colonies left in the bag, so the probability that the second student selects Virginia is 1/12.

To find the probability that both events occur, we multiply the probabilities:

(1/13) x (1/12) = 1/156

To express this probability as a percentage, we multiply by 100:

(1/156) x 100 ≈ 0.64%

Therefore, the probability that the first student selects Pennsylvania and the second student selects Virginia is approximately 0.64%.

More on probability can be found here: https://brainly.com/question/30034780

#SPJ1

Solve for m. m/2 - squareroot 5 = 3

Answers

Answer:

We can start by isolating the variable term on one side of the equation and moving all other terms to the other side.

First, we'll add the square root of 5 to both sides of the equation:

m/2 - sqrt(5) + sqrt(5) = 3 + sqrt(5)

Simplifying:

m/2 = 3 + sqrt(5)

Next, we'll multiply both sides of the equation by 2:

m = 2(3 + sqrt(5))

Simplifying:

m = 6 + 2sqrt(5)

Therefore, the value of m is 6 + 2sqrt(5).

Step-by-step explanation:

Forty people were asked their favorite kind of pizza. Thirty percent of the people surveyed chose sausage. How many people preferred sausage?

Answers

To find out how many people preferred sausage, you can start by multiplying the percentage who chose sausage, which is 30%, by the total number of people surveyed, which is 40.

So, 30% of 40 is:

0.30 x 40 = 12

Therefore, 12 people preferred sausage.

Answer: 12 people

Step-by-step explanation:

0.30 x 40 = 12

Alice finds shirts on sale for $18.99.She buys twelve how much money does she spend?

Answers

Answer:

Well, if Alice buys 12 shirts which each cost $18.99 the equation would be 18.99 * 12 which = 227.88

Alice spent $227.88 on 12 shirts

Step-by-step explanation:

Answer:

Well, if Alice buys 12 shirts which each cost $18.99 the equation would be 18.99 * 12 which = 227.88

Step-by-step explanation:

The following blueprint of a kitchen has dimensions of 7 inches by 7 inches. The island has been highlighted in red.


The island's actual dimensions are 3 1/2 feet by 1 3/4 feet. If the scale of the blueprint is 1 inch = 2 feet, what are the dimensions of the island on the blueprint?

Answers

The dimensions of the island on the blueprint are 14 inches by 3.5 inches.

We have,

The actual dimensions of the island are 3 1/2 feet by 1 3/4 feet.

We need to find the dimensions of the island on the blueprint, given that the scale of the blueprint is 1 inch = 2 feet.

To convert the actual dimensions to the dimensions on the blueprint, we need to use the scale factor of 1 inch = 2 feet.

We can set up a proportion to relate the actual dimensions to the dimensions on the blueprint:

Actual dimension/blueprint dimension = scale factor

Let x be the length of the island on the blueprint.

Then we can set up the following proportion:

3.5 feet / (1.75 feet)

= x inches / 7 inches

Simplifying,

2 = x / 7

Multiplying both sides by 7, we get:

x = 14 inches

The length of the island on the blueprint is 14 inches.

Similarly, we can find the width of the island on the blueprint:

1.75 feet / 3.5 feet

= y inches / 7 inches

Simplifying, we have:

0.5 = y / 7

Multiplying both sides by 7, we get:

y = 3.5 inches

The width of the island on the blueprint is 3.5 inches.

Thus,

The dimensions of the island on the blueprint are 14 inches by 3.5 inches.

Learn more about expressions here:

https://brainly.com/question/3118662

#SPJ1

Is this right. Don’t know the difference between the stratified and systematic random. Pic below

Answers

The difference between the stratified and systematic random sampling is explained below.

Systematic random sampling is a method where every Kth person of the population is chosen to be part of the sample, whereas stratified random sampling is a method where the population is first divided into subgroups, and then drawing a simple random sample from each subgroup.

Learn more about stratified and systematic random sampling click;

https://brainly.com/question/28963265

#SPJ1

Find the value of the following expression and round to the nearest integer:

Answers

The value of the Expression is 610, 919.

We have,

Expression : [tex]\sum_{n=0}^{61[/tex] 700 (1.07[tex])^{n+1[/tex]

The expression is a summation formula, representing the sum of a series of values.

Here r= 1.07 and n is number of terms.

So, [tex]\sum_{n=0}^{61[/tex] 700 (1.07[tex])^{n+1[/tex]

= 700 (1) +  [tex]\sum_{n=0}^{61[/tex] 700 (1.07[tex])^{n[/tex]

= 700 + 700 ([tex]r^{61[/tex] - 1)/ (r-1)

= 700 + 700 ([tex](1.07)^{61[/tex]-1)/ (0.07)

= 700+ 700 (871.7428)

= 610, 919

Learn more about Summation here:

https://brainly.com/question/29103782

#SPJ1

construct a binomial whose gcf is 7a^3

Answers

The binomial whose greatest common factor is 7a³ is 14a⁴ + 35a³.

Given, the a³ is not common in A and C.

Now, in 14a⁴ + 35a³

14a⁴ = 2 x 7 x a³ x a

35a³ = 7 x 5 x a³

So, the common factors of 14 and 35 is 7.

Then, 14a⁴ + 35a³

= ( 2 x 7 x a³ x a) + (  5 x 7 x a³)

= 7a³ (2a + 5)

Learn more about greatest common factor here:

brainly.com/question/11221202

#SPJ1

Helppp I need the ending numbers ?

Answers

The solution is, the simplification of the equation is :

4x^2 - 4 = (2x -2 ) (2x+2)

Here, we have,

given that,

the expression is:

4x^2 - 4

now, we have to simplify this.

we know that ,

The a^2 - b^2 formula is also known as "the difference of squares formula". The a square minus b square is used to find the difference between the two squares without actually calculating the squares.

It is one of the algebraic identities.

It is used to factorize the binomials of squares.

The a^2 - b^2 formula is given as:

a^2 - b^2 = (a - b) (a + b).

so, we have,

4x^2 - 4

= (2x)^2 - 2^2

= (2x -2 ) (2x+2)

Hence, The solution is, the simplification of the equation is :

4x^2 - 4 = (2x -2 ) (2x+2)

To learn more on Equation:

brainly.com/question/10413253

#SPJ1

You can use the notation P(A), read “the probability of event B, given event A” to write a

A. Probability distribution
B. Frequency table
C. Conditional probability
D. Cumulative probability

Answers

You can use the notation P(A), read “the probability of event B, given event A” to write a conditional probability. The correct answer is C.

Conditional probability refers to the probability of one event occurring given that another event has already occurred. In this case, we are interested in the probability of event B occurring given that event A has already occurred, and we can represent this using the notation P(B|A), where '|' means 'given'.

For example, let's say we are interested in the probability of getting a head on a coin toss (event B), given that the coin was flipped and landed on heads (event A). We could represent this using the notation P(B|A). The value of P(B|A) would be 1, because if the coin already landed on heads, then the probability of getting a head on the next flip is certain.

Conditional probability is an important concept in probability theory and is often used in real-world applications, such as predicting the likelihood of a disease given certain symptoms, or the probability of an event occurring given certain conditions.

The correct answer is C.

To learn more about probability click on,

https://brainly.com/question/29259732

#SPJ1

What is the value of z?

Answers

Answer: z = 8

Step-by-step explanation:

The diagram shows us that 8z + 3z + 2 = 90

so we can say that: 11z = 88

therefore z = 8.

Note: diagrams can be misleading! this diagram technically shows us that 64 < 26!

how do you find the net of a rectangular prism

Answers

Answer:

The formula looks like this:

Surface Area = 2 l w + 2 l h + 2 h w ,

where SA = surface area, l = length, w = width, and h = height. In the rectangular prism net above, l = 8 inches, w = 5 inches, and h = 3 inches. Simply put these numbers into the formula and solve for surface area.

A curve, described by x2 + y2 + 6y = 0, has a point A at (−3, −3) on the curve.

Part A: What are the polar coordinates of A? Give an exact answer.

Part B: What is the polar form of the equation? What type of polar curve is this?

Part C: What is the directed distance when theta equals 4 pi over 3 question mark Give an exact answer.

Answers

a) The polar coordinates of point A are (√(18), π/4).

b) The curve is a circle centered at the origin with radius 6.

c) The directed distance is the value of r, which is 6 √(3).

To find the polar coordinates of point A on the curve, we need to convert the point from Cartesian to polar coordinates. The conversion formula is:

r = √(x² + y²)

θ = arctan(y/x)

Using the values of point A, we have:

r = √((-3)² + (-3)²) = √(18)

θ = arctan((-3)/(-3)) = arctan(1) = π/4

To find the polar form of the equation x² + y² + 6y = 0, we need to convert it from Cartesian to polar coordinates. The conversion formulae are:

x = r cos(θ)

y = r sin(θ)

Using these formulae, we can rewrite the equation as:

r² cos²(θ) + r² sin²(θ) + 6r sin(θ) = 0

Simplifying this equation, we get:

r = -6 sin(θ) / (1 - cos²(θ))

To find the directed distance when θ equals 4 π over 3, we need to substitute this value of θ into the polar equation we found in Part B. Doing so, we get:

r = -6 sin(4 π/3) / (1 - cos²(4 π/3))

r = -6(-√(3)/2) / (1 - (-1/2)²)

r = 6 √(3)

To know more about polar coordinates here

https://brainly.com/question/31422978

#SPJ1

A belt runs a pulley at 80 revolutions per minute. Find the angular velocity of the pulley in radians per second.

Answers

A belt runs a pulley at 80 revolutions per minute.

(a) To find the angular speed of the pulley in radians per second, we can use the formula

ω = 2πf

Where

ω = angular speed in radians per second

f = frequency in Hertz

We know that the pulley rotates at a rate of 80 revolutions per minute. To convert this to a frequency in Hertz, we can divide by 60 seconds per minute

f = 80 rev/min ÷ 60 min/s = 4/3 Hz

Now we can plug in this value for f into the formula

ω = 2πf = 2π(4/3) = 8π/3 rad/sec

Therefore, the angular speed of the pulley is 8π/3 radians per second.

(b) To find the linear speed of the belt in centimeters per second, we can use the formula

v = rω

Where

v = linear speed in centimeters per second

r = radius of the pulley in centimeters

ω = angular speed in radians per second

We know the radius of the pulley is given in centimeters. Let's assume it is r cm. We just found the angular speed to be 8π/3 radians per second. Now we can plug in these values into the formula

v = rω = r(8π/3) = (8π/3)r cm/s

Therefore, the linear speed of the belt is (8π/3)r centimeters per second.

The given question is incomplete and the complete question is ''A belt runs a pulley at 80 revolutions per minute. Find the angular velocity of the pulley in radians per second. (b)Find the linear speed of the belt in centimeters per second''.

To know more about revolutions here

https://brainly.com/question/29707118

#SPJ1

Give the rectangular coordinates for the point

Answers

The rectangular coordinates from the polar coordinates are: (-4√2, -4√2)

How to convert polar coordinates to rectangular coordinates?

The steps to  to convert polar coordinates to rectangular coordinates are:

Step 1: Find the x -coordinate for the rectangular coordinate form of the point by using the equation x = r cos(θ)

Step 2: Find the y -coordinate for the rectangular coordinate form of the point by using the equation y = rsin(θ)

Step 3: Write the rectangular coordinates as (x,y) using the results from steps 1 and 2.

We are given the polar coordinates as (8, 225°)

Thus:

x-coordinate of rectangular coordinate = 8 cos 225 = -4√2

y-coordinate of rectangular coordinate = 8 sin 225 = -4√2

Read more about rectangular coordinates at: https://brainly.com/question/29155884

#SPJ1

An insurance company offers an ordinary annuity that earns 6.5% interest compounded annually. A couple plans to make equal annual deposits into this account for 30 years and then make 20 equal annual withdrawals of €25,000, reducing the balance of the account to zero.

(i) Compute the value of the fund based on the withdrawals required. [5 marks]

(ii) Compute the amount of each deposit needed in order to maintain the fund. [5 marks]

(iii) Compute the total interest earned over the entire 50 years. [5 marks]​

Answers

Answer:

(i) To compute the value of the fund based on the withdrawals required, we can use the formula for the future value of an annuity due:

FV = P * ((1 + r)^n - 1) / r) * (1 + r)

where FV is the future value of the annuity, P is the annual payment, r is the interest rate per period, n is the total number of periods, and the extra (1 + r) factor is because the payments are made at the beginning of each period.

In this case, P = €25,000, r = 0.065, n = 20. We want to find the future value at the end of the 20-year period:

FV = 25000 * ((1 + 0.065)^20 - 1) / 0.065) * (1 + 0.065)

FV ≈ €743,704.96

Therefore, the value of the fund based on the withdrawals required is approximately €743,704.96.

(ii) To compute the amount of each deposit needed in order to maintain the fund, we can use the formula for the present value of an ordinary annuity:

PV = P * ((1 - (1 + r)^(-n)) / r)

where PV is the present value of the annuity, P is the annual payment, r is the interest rate per period, and n is the total number of periods.

In this case, PV = €743,704.96, r = 0.065, n = 20. We want to find the annual payment:

PV = P * ((1 - (1 + 0.065)^(-20)) / 0.065)

P ≈ €22,630.53

Therefore, the amount of each deposit needed in order to maintain the fund is approximately €22,630.53.

(iii) To compute the total interest earned over the entire 50 years, we can subtract the total deposits from the total withdrawals, and then subtract the initial balance. The total deposits are the annual deposit amount times the number of years (30), and the total withdrawals are the annual withdrawal amount times the number of years (20). The initial balance is the present value of the annuity that we calculated in part (ii).

Total deposits = €22,630.53 * 30 = €678,915.90

Total withdrawals = €25,000 * 20 = €500,000

Initial balance = €743,704.96

Total interest earned = Total withdrawals - Total deposits - Initial balance

Total interest earned = €500,000 - €678,915.90 - €743,704.96

Total interest earned ≈ -€922,620.86

Note that the negative sign indicates that the insurance company actually earned interest on this annuity, rather than the couple earning interest on their investment. This is because the withdrawals are greater than the deposits, and the interest rate earned by the insurance company is greater than the interest rate paid to the couple.

Step-by-step explanation:

Other Questions
What is the best definition for hypothermia? Damage to skin caused by long exposure to freezing temperatures Very low internal body temperature caused by cold temperatures Significantly increased heart rate caused by cold temperatures Elevated blood pressure caused by vigorous exercise Mrs. Slayton was told by her physician to pick up generic Cortizone cream. Which medication name will be labeled on the package? Betamethasone Clobetasol Hydrocortisone Mometasone Notice that the bond strength for lithium bonded with any of the anions is larger than the bond strength of potassium bonded with any of the same anions. Propose a scientifically sound explanation for this. When installing post- tension cables, all of the following practices are acceptable except Select the correct answer. Which point lies on the circle represented by the equation (x 3)2 + (y + 4)2 = 62? A. (9,-2) B. (0,11) C. (3,10) D. (-9,4) E. (-3,-4) Reset Next How many solutions does the equation sin 3x=. 25-x^2 have? Use Newton's method to find them-answer in whole number what could happen if heat fixation was performed prior to doing a capsular stain The median of the data set is 18. What number is missing? 12,17,__,21,13,25 What disease is diagnosed by refusal to maintain normal body weight intense fear of weight gain fear of fat disturbance of body image and selfstarvation? Why does sustainable forest management matter to society?a. People have somewhere to visit on weekends.b. It makes people feel good about paying taxes.c. Healthy forests work to increase oxygen in the air.d. It regulates forest resources to meet society needs. What is 8. 19 divided by 4. 2 and show your work which of the following are water-conducting cells that are dead at functional maturity?a) parenchyma cellsb) collenchyma cellsc) tracheids and vessel elementsd) sieve-tube elements which of the following statements about the characteristics of minerals are correct? 1.) minerals will have the same streak color 2.) a mineral with a higher hardness value will scratch one with a lower value 3.) metallic minerals will usually have a shiny luster 4.) minerals with cleavage will split in clean cuts without jagged edges answers: 2,3 and 4 only or 1,2 and 3 only or 1,3 or 4 only or 1, 2 and 4 only 6. A firm has forecasted sales of $3,000 in April, $4,500 in May, and $6,500 in June. All sales are on credit. 30% is collected the month of sale and the remainder the following month. What will be the balance in accounts receivable at the end of June? A. $1,950B. $6,500C. $4,550D. $5,100 The 1987 UN Commission on Sustainability first introduced the concept of ________ as a necessary focus for maintaining sustainability.A) human well-beingB) renewable energyC) environmental sustainabilityD) cultural servicesE) human population growth Camber Company habcnt Account that control account of 510 500 on January 1, 2022. The bary ledger contain the counts Border Company balance 53000, Cuber Compact 1.000 and Free Company Ouring January the following reciented transaction Gredit Sales Collection de Company $9.200 $8,100 50 Cuddyer Company 7.000 2500 2.300 Free Company 3.200 9.200 0 What is the January 1 balance in the Free Company Subsidiary account? Balance in the Freeze Company Subsidiary account 5 e Textbook and Media What is the January 31 balance in the control account? Batang Glance in the control account Textbook and Media Compute the balances in the subsidiary accounts at the end of the month Bader $ Cuddyer 5 Free $ RTextbook and Media Which January transaction would not be recorded in a specialjoumal the principle that sacred things are to be touched so that power may be transferred refers to which of the following minimal categories of religion?a. prayerb. sacrificec. manad. taboo Benefits of being a member of a common market, or "trading bloc" include: one of the factors determining the speed and duration of a twitch in cardiac muscles fibers is the speed of the Two point charges each carrying a charge of +3.5 E6 C are located 3.5 meters away from each other.How strong is the electrostatic force between the two points (k = 9.0 E9 Nm2/C2)?Is this force a repulsive force or an attractive force?Remember to identify all data (givens and unknowns), list equations used, show all your work, and include units and the proper number of significant digits to receive full credit.